Nothing Special   »   [go: up one dir, main page]

Hand OITE - 2012 2013 2014

Download as ppt, pdf, or txt
Download as ppt, pdf, or txt
You are on page 1of 209

HAND

University of Pennsylvania Department of Orthopaedic Surgery


2012-01 (Hand)

• For an otherwise healthy carpenter with


symptoms of dominant-hand ulnar-sided
pain, hand pallor, and coolness, the
preliminary work-up should include?

• 1. a CT scan.
• 2. a MRI scan.
• 3. a bone scan.
• 4. sonography.
• 5. angiography.
2012-01 (Hand)

• For an otherwise healthy carpenter with


symptoms of dominant-hand ulnar-sided
pain, hand pallor, and coolness, the
preliminary work-up should include?
•  
• 1. a CT scan.
• 2. a MRI scan.
• 3. a bone scan.
• 4. sonography.
• 5. angiography.
2012-01 (Hand)

• The description of a manual laborer with ulnar-sided hand


pain and ischemic symptoms is classic for hypothenar
hammer syndrome. This results from repetitive trauma
when using the hypothenar pad as a ‘hammer’, which
causes the superficial arch of the ulnar artery to be
compressed against the hook of the hamate. This in turn
leads to aneurysm formation and segmental occlusion of
the artery. The first modality for diagnosis should be
noninvasive, such as sonography. U/S also helps
determine if an aneurysm is present. Angiography is the
gold-standard but more invasive and does not diagnose
aneuryms as reliably.
2012-02 (Hand)

• A 42-year-old patient with a right distal


radius fracture underwent open reduction and
internal fixation. To reduce the likelihood of
complex regional pain syndrome, the most
appropriate medication is?

• 1. Biotin.
• 2. Tramadol.
• 3. vitamin A.
• 4. vitamin C.
• 5. vitamin E.
2012-02 (Hand)

• A 42-year-old patient with a right distal


radius fracture underwent open reduction and
internal fixation. To reduce the likelihood of
complex regional pain syndrome, the most
appropriate medication is?

• 1. Biotin.
• 2. Tramadol.
• 3. vitamin A.
• 4. vitamin C.
• 5. vitamin E.
2012-02 (Hand)

• Vitamin C has shown efficacy in reducing the incidence of


CRPS after distal radius fractures in large, well-designed
clinical trials. The mechanism is not fully understood but it
may work by modulating the inflammatory signals
contributing to CRPS. Anyone discharging a distal radius
fracture from the hand service should have picked this one
up as it has become standard protocol.
2012-15 (Hand)

• A contraindication for attempting a


replantation by an experienced surgeon in an
appropriately equipped facility includes
– 1. Amputation through flexor zone I.
– 2. Major limb amputation in a child.
– 3. Absence of athrosclerotic disease.
– 4. Crush or avulsion mechanism of amputation.
– 5. Sharp transection of the thumb at the metatarsophalangeal joint
level.
Question 15

• A contraindication for attempting a


replantation by an experienced surgeon in an
appropriately equipped facility includes
– 1. Amputation through flexor zone I.
– 2. Major limb amputation in a child.
– 3. Absence of athrosclerotic disease.
– 4. Crush or avulsion mechanism of amputation.
– 5. Sharp transection of the thumb at the metatarsophalangeal joint
level.
Question 15

• Primary indications for replantation


– Thumb
– Multiple digits
– Wrist Level or proximal
– Any amputation in a child
• Relative indications is a level distal to the FDS insertion (Zone 1)
• Primary contraindications to replanation
– Single digit amputation, especially index
– Crushed or mangled extremities
– Prolonged ischemia
– Segmental amputations
– Level of amputation within Zone II of the flexor tendon sheath
2012-26 (Hand)

• Figure 26 is the clinical photograph of a 12-year-old boy with a


learning disability who was riding a bicycle without wearing gloves
for about 10 minutes when the temperature was 10°F. He arrived
at the emergency department with painful fingers that were warm
and perfused without ischemia. Which of the following treatments
is recommended?
 
• 1. Escharotomy
• 2. Surgical debridement of the blisters
• 3. Surgical debridement of the blisters and skin grafting
• 4. Administration of subcutaneous calcium gluconate
• 5. Rewarming of the fingers in a warm water bath at 104°F
Q26
Q26

• Figure 26 is the clinical photograph of a 12-year-old boy with a


learning disability who was riding a bicycle without wearing gloves
for about 10 minutes when the temperature was 10°F. He arrived
at the emergency department with painful fingers that were warm
and perfused without ischemia. Which of the following treatments
is recommended?
 
• 1. Escharotomy
• 2. Surgical debridement of the blisters
• 3. Surgical debridement of the blisters and skin grafting
• 4. Administration of subcutaneous calcium gluconate
• 5. Rewarming of the fingers in a warm water bath at 104°F
Q26

• Rewarming for frostbite should not begin until definitive medical


care can be provided to avoid repeated freeze-thaw cycles, as
these cause additional tissue necrosis. Rewarming should be rapid
and for an affected limb should be performed by submersion in
warm water at 104 degrees to 107.6 degrees F (40 degrees to 42
degrees C) for 15 to 30 minutes. Débridement of necrotic tissues
is generally delayed until there is a clear demarcation from viable
tissues, a process that usually takes from 1 to 3 months from the
time of initial exposure. Immediate escharotomy and/or
fasciotomy is necessary when circulation is compromised.
2012-38 (Hand)
Figure 38 is the sagittal CT scan of a 45-year-old woman who injured her wrist after a
fall. Appropriate treatment of the fracture should include?
 
1. cast immobilization.
2. closed reduction and percutaneous pin fixation.
3. open reduction and volar locking plate fixation.
4. open reduction and radial styloid column plating.
5. open reduction and internal fixation of the volar fracture fragment.
5. Open reduction and internal fixation of the volar fracture fragment.

The stability of comminuted fractures of the distal part of the radius with volar
fragmentation is determined not only by the reduction of the major fragments but
also by the reduction of the small volar lunate fragment. The distal volar lunate
fragment is the site of origin of the strong volar radiolunate ligaments which insert
onto the lunate, and so displacement of this small piece volarly will allow the
lunate and the rest of the carpus to subluxate volarly. The unique anatomy of this
fragment may prevent standard fixation devices for distal radial fractures from
supporting the entire volar surface effectively, as a standard volar plate cannot
capture this small distal piece without risking injury to the flexor tendons.
Fragment specific fixation of the volar lunate facet fragment with commercially
available small plates, or with a tension-band construct or augmentation with K-
wires may be required to reduce and stabilize this fragment.

Harness et al. reported on a cohort of 7 patients with a volar shearing fracture of


the distal radius who lost fixation of a volar lunate facet fragment with
subsequent carpal displacement after open reduction and internal fixation. Five
patients underwent revision surgery with adequate results.
2012-49 (Hand)

• Figure 49 features the radiographs of a 22-


year-old man. The most appropriate
treatment is?

• 1. observation.
• 2. cast treatment.
• 3. buddy tape and active motion.
• 4. closed reduction and pinning.
• 5. open reduction and internal fixation.
Question 49: Images
Question 49: Answer

• The correct response is 4- closed reduction


and pinning.
Question 49: Explanation
• The fracture is a proximal phalanx fracture of the small finger.
It is displaced, but is a stable pattern. These fractures require
reduction to avoid the hyperextension deformity associated
with fractures in this location. Because it will be hard to hold
with immobilization, two pins can be placed to hold the
alignment for 3-4 weeks and then pulled in the office. More
comminuted fractures of the proximal phalanx would require
ORIF.

• RECOMMENDED READINGS:
• Henry MH. Fractures of the proximal phalanx and metacarpals
in the hand: preferred methods of stabilization. J Am Acad
Orthop Surg. 2008 Oct;16(10):586-95. Review. PubMed PMID:
18832602.
• Slade JF III, Oetgen ME. Phalangeal injuries. In: Trumble TE,
Budoff JE, eds. Hand Surgery Update IV. Rosemont, IL:
American Society for Surgery of the Hand; 2007:3-25.
2012-64 (Hand)
Figure 64 is a T2-weighted MRI scan of a 64-year-old man who has had a right
volar radial mass for the past 2 years. What is the most likely diagnosis?
 
1. Lipoma
2. Ganglion
3. Schwannoma
4. Radial artery aneurysm
5. Giant-cell tumor of tendon sheath
Question 64

University of Pennsylvania Department of Orthopaedic Surgery


Question 64

Figure 64 is a T2-weighted MRI scan of a 64-year-old man who has had a right
volar radial mass for the past 2 years. What is the most likely diagnosis?
 
1. Lipoma
2. Ganglion
3. Schwannoma
4. Radial artery aneurysm
5. Giant-cell tumor of tendon sheath

RECOMMENDED READINGS:
Nahra ME, Bucchieri JS. Ganglion cysts and other tumor related conditions of
the hand and wrist. Hand Clin. 2004 Aug;20(3):249-60, v. Review. PubMed
PMID: 15275684.

Peh WC, Truong NP, Totty WG, Gilula LA. Pictorial review: magnetic resonance
imaging of benign soft tissue masses of the hand and wrist. Clin Radiol. 1995
Aug;50(8):519-25. Review. PubMed PMID: 7656517.

University of Pennsylvania Department of Orthopaedic Surgery


Question 64

Figure 64 is a T2-weighted MRI scan of a 64-year-old man who has had a right
volar radial mass for the past 2 years. What is the most likely diagnosis?
 
1. Lipoma
2. Ganglion
3. Schwannoma
4. Radial artery aneurysm
5. Giant-cell tumor of tendon sheath

EXPLANATION:
Ganglia are probably the commonest soft tissue masses found in the hand and
wrist arising from joints, tendons or tendon sheaths, especially involving
the extensor surfaces. On T1-weighted images, signal intensity of
ganglia is either isointense or slightly hyperintense compared to muscle. On T2-
weighted images, signal is typically hyperintense. Ganglia are usually
homogeneous on all sequences, have well-defined margins, and may be
contiguous with adjacent joint capsules or tendon sheaths. The image provided
is a T2 image.

University of Pennsylvania Department of Orthopaedic Surgery


2012-85 (Hand)

• Figures 85a and 85b are the radiographs of a 20-


year-old man who injured his finger after being
hit with a baseball. Treatment should be?
– 1 – Early motion as tolerated
– 2 – Surgical reduction and pinning
– 3 – Distal interphalangeal joint fusion
– 4 – Splinting of the distal interphalangeal joint in
extension
– 5 – Splinting of the proximal interphalangeal joint in
extension
Question 85
Question 85

• Figures 85a and 85b are the radiographs of a 20-


year-old man who injured his finger after being
hit with a baseball. Treatment should be?
– 1 – Early motion as tolerated
– 2 – Surgical reduction and pinning
– 3 – Distal interphalangeal joint fusion
– 4 – Splinting of the distal interphalangeal joint in
extension
– 5 – Splinting of the proximal interphalangeal joint in
extension
Question 85

• The injury is a bony avulsion of the EDP, i.e.


a Mallet Finger. The bone fragment is
retracted and will not heal in this position.
Non-operative management is extension
splinting in fractures involving less than
30% of the articular surface and without
evidence of DIPJ subluxation. In this case
there are no injury films to show if the joint
was subluxed, but there is a large bony
fragment that has avulsed off the dorsal
base of the distal phalanx. The correct
answer choice would be to operatively fix
this fracture using the Ishiguro Extension
Block Technique depicted on the right.
2012-89 (Hand)

Figures 89a through 89c are the radiographs of a 66-


year-old man who fell 1 week ago. Examination
revealed rotational deformity of his index finger.
Treatment should consist of reduction and?

• 1. casting.
• 2. buddy taping.
• 3. plate fixation.
• 4. intramedullary fixation.
• 5. interfragmentary fixation.

University of Pennsylvania Department of Orthopaedic Surgery


University of Pennsylvania Department of Orthopaedic Surgery
Question 89
Figures 89a through 89c are the radiographs of a 66-
year-old man who fell 1 week ago. Examination
revealed rotational deformity of his index finger.
Treatment should consist of reduction and?

• 1. casting.
• 2. buddy taping.
• 3. plate fixation.
• 4. intramedullary fixation.
• 5. interfragmentary fixation.

University of Pennsylvania Department of Orthopaedic Surgery


Explanation
For long oblique metacarpal fractures, lag screw technique is the best
method, because this technique provides sufficient rigidity to allow early
mobilization, with little tendency to affect tendon gliding.

RECOMMENDED READINGS:

Kawamura K, Chung KC. Fixation choices for closed simple unstable


oblique phalangeal and metacarpal fractures. Hand Clin. 2006
Aug;22(3):287-95. Review. PubMed PMID: 16843795. Henry MH.
Fractures of the proximal phalanx and metacarpals in the hand:
preferred methods of stabilization. J Am Acad Orthop Surg. 2008
Oct;16(10):586- 95. Review. PubMed PMID: 18832602.

Freeland AE, Orbay JL. Extraarticular hand fractures in adults: a review


of new developments. Clin Orthop Relat Res. 2006 Apr;445:133-45.
Review. PubMed PMID: 16505726.

University of Pennsylvania Department of Orthopaedic Surgery


2012-98 (Hand)
• Which wrist arthroscopy portal places a subcutaneous sensory nerve at
most risk is?

• 1. 1-2.
• 2. 3-4.
• 3. 4-5.
• 4. ulnar midcarpal.
• 5. radial midcarpal.
Question 98
• Which wrist arthroscopy portal places a subcutaneous sensory nerve at
most risk is?

• 1. 1-2.
• 2. 3-4.
• 3. 4-5.
• 4. ulnar midcarpal.
• 5. radial midcarpal.
Question 98
• The 1-2 wrist arthroscopy portal places the superficial branch of the radial nerve
(SBRN) at risk and is the closest in proximity to any sensory nerve compared to the
other portals listed in the question stem

• The 1-2 portal is placed between the ECRB and APL. Care must be taken when
accessing this portal due to reported injuries to the superficial branch of the radial
nerve as well as radial artery. 

• Kilic et al. dissected 6 cadavers to determine the course of the SBRN. They found that
it was .9mm from the 1-2 portal at its closest, and that care should be taken when
creating this portal given frequent variations in course. 

• Auerbach et al. dissected 20 cadavers to determine the course of the SBRN. In all
specimens, the nerve arose between the brachioradialis and ECRL ~8cm proximal to
the radial styloid. On average, 5.8 branches of the SBRN crossed the wrist joint. They
found some variability within the course, and urge surgeons to be respectful of the
nerve during dissection.

• Tryfonidis et al. performed a cadaveric study to determine the anatomic relation of


dorsal wrist arthroscopy portals and superficial nerves. They found that the 1-2 portal
was only 1.82 mm from the SBRN. Compared to the 3-4 portal, 4-5 portal, 6U portal
and midcarpal portal, the 1-2 portal was the closest in proximity to any sensory
nerve.
2012-135 (Hand)
Figures 135a through 135c are the radiograph and clinical
photographs of a 15-year-old left-hand dominant boy who amputated
his left hand through the midcarpal joint with a saw. The hand and
patient arrived within 1 hour of the injury. The hand was wrapped in
a moist saline dressing, put in a plastic bag, and placed in a cooler on
top of ice. The next treatment step should include?

1. replantation of the hand


2. free-flap coverage of the wound
3. radial artery flap coverage of the wound
4. revision of the amputation wound to a distal forearm amputation
5. revision of the amputation at the wrist to preserve the distal
radioulnar joint

University of Pennsylvania Department of Orthopaedic Surgery


Question 135

University of Pennsylvania Department of Orthopaedic Surgery


Question 135

Figures 135a through 135c are the radiograph and clinical


photographs of a 15-year-old left-hand dominant boy who amputated
his left hand through the midcarpal joint with a saw. The hand and
patient arrived within 1 hour of the injury. The hand was wrapped in
a moist saline dressing, put in a plastic bag, and placed in a cooler on
top of ice. The next treatment step should include?

1. replantation of the hand


2. free-flap coverage of the wound
3. radial artery flap coverage of the wound
4. revision of the amputation wound to a distal forearm amputation
5. revision of the amputation at the wrist to preserve the distal
radioulnar joint

University of Pennsylvania Department of Orthopaedic Surgery


Question 135

1. replantation of the hand

The decision to replant is based on the determination that the anticipated function and
overall well-being of the patient after replantation will be better than that after revision
amputation.

Indications for distal upper extremity replantation are expanding. Traditionally,


replantation is recommended for amputation of the thumb at all levels, multiple digits,
amputations in children, and single finger amputations distal to the flexor digitorum
superficialis insertion. Transmetacarpal-level amputations, as in this clinical
scenario, pose a unique challenge to the surgeon owing to the increased complexity of
the procedure and the number of structures involved. However, successful results have
been reported, and should be attempted (especially in the setting of this young patient,
who presents within one hour after sharply severing severing his dominant hand).

RECOMMENDED READINGS:
Sabapathy SR, Venkatramani H, Bharathi RR, Bhardwaj P. Replantation surgery. J Hand Surg Am. 2011 Jun;36(6):1104-10.
PubMed PMID: 21636026.
Hanel DP, Chin SH. Wrist level and proximal-upper extremity replantation. Hand Clin. 2007 Feb;23(1):13-21. Review.
PubMed PMID: 17478249.

University of Pennsylvania Department of Orthopaedic Surgery


2012-147 (Hand)

A 28-year-old man sustained the injury seen in Figures


147a and 147b. During surgery, after radius fixation, the
distal radioulnar joint was evaluated and instability was
noted. The next step should be?

1. casting in pronation.
2. casting in supination.
3. early range of motion.
4. ligament reconstruction using a tendon graft.
5. percutaneous fixation of the distal radioulnar joint.
Question 147
Question 147

A 28-year-old man sustained the injury seen in Figures


147a and 147b. During surgery, after radius fixation, the
distal radioulnar joint was evaluated and instability was
noted. The next step should be?

1. casting in pronation.
2. casting in supination.
3. early range of motion.
4. ligament reconstruction using a tendon graft.
5. percutaneous fixation of the distal radioulnar joint.
Question 147

Figure 147 shows a distal radial shaft fracture. As in the case, this
fracture is often associated with a distal radioulnar joint (DRUJ)
dislocation, termed Galeazzi fracture. In Korompilias’s (2011)
retrospective review of 95 patients, residual DRUJ instability after radial
shaft fixation was most commonly found in Type I fractures (distal third
shaft fractures) compared to Type II (middle third) or Type III (proximal
third). Residual DRUJ instability was treated with 4 weeks temporary
stabilization with a single K-wire and long-arm casting for 6 weeks. At a
mean follow-up of 6.8 years, none of the patients required further DRUJ
stabilization, had persistent DRUJ instability or needed further surgery.
Question 147

RECOMMENDED READINGS:

Korompilias AV, Lykissas MG, Kostas-Agnantis IP, Beris


AE, Soucacos PN. Distal radioulnar joint instability
(Galeazzi type injury) after internal fixation in relation to
the radius fracture pattern. J Hand Surg Am. 2011
May;36(5):847-52. Epub 2011 Mar 23. PubMed PMID:
21435802.

Giannoulis FS, Sotereanos DG. Galeazzi fractures and


dislocations. Hand Clin. 2007 May;23(2):153-63, v.
Review. PubMed PMID: 17548007.

Rettig ME, Raskin KB. Galeazzi fracture-dislocation: a


new treatment-oriented classification. J Hand Surg
2012-148 (Hand)

Coverage in a patient who sustains a thumb-tip injury


with 2.5-cm pulp skin loss is best achieved with?

1. skin graft.
2. direct closure.
3. free soft-tissue transfer.
4. volar advancement flap.
5. island volar advancement flap.
Question 148

Coverage in a patient who sustains a thumb-tip injury


with 2.5-cm pulp skin loss is best achieved with?

1. skin graft.
2. direct closure.
3. free soft-tissue transfer.
4. volar advancement flap.
5. island volar advancement flap.
Question 148

A “reconstruction ladder” is helpful in determining the least invasive procedure


to obtain the optimal outcome. The ladder includes primary closure, healing by
secondary intention, split-thickness skin grafts, full-thickness skin grafts,
random pattern local flaps, axial pattern local flaps, island patter local flaps,
distant random pattern flaps, distant axial pattern flaps, and free tissue
transfer.

In 1964, Moberg described a palmar advancement flap for pulp reconstruction


in the treatment of partial or complete amputations of the thumb. The defect
size that can be safely covered varies in the literature from 1.8 to 3cm.
However, the mobility of this flap is limited becaseu of the stiff nature of the
thumb skin and underlying fascial connections.

Mutaf et al (2011) describe the use of the volar advancement flap of the thumb
as a pure island flap in which all proximal attachments with the exception of the
neurovascular bundles are divided to provide maximum advancement. The flaps
healed uneventfully in all patients in their small case series with minimum 2
year follow-up.
Question 148

RECOMMENDED READINGS:

Foucher G, Delaere O, Citron N, Molderez A. Long-term


outcome of neurovascular palmar advancement flaps for
distal thumb injuries. Br J Plast Surg. 1999 Jan;52(1):64-
8. PubMed PMID: 10343593.

Baumeister S, Menke H, Wittemann M, Germann G.


Functional outcome after the Moberg advancement flap in
the thumb. J Hand Surg Am. 2002 Jan;27(1):105-14.
PubMed PMID: 11810623.

Mutaf M, Temel M, Gunal E, Isik D. Island volar


advancement flap for reconstruction of the thumb defects.
Ann Plast Surg. 2011 May 27. PubMed PMID: 21629080.
2012-158 (Hand)
• Figures 158a through 158c are the radiographs of a 38-year-
old man with a previous history of finger and hand infection
treated with amputation 2 years ago. While there has been
no subsequent history of redness or drainage, he now has
difficulty using his hand, especially when grasping objects
such as coins. Examination revealed a gap between his index
and ring fingers, and the index finger rotated toward the
ring. Treatment should consist of?

1. cosmetic prosthetic fitting.


2. functional prosthetic fitting.
3. index metacarpal transposition.
4. metacarpal derotational osteotomy.
5. free toe-to-hand microvascular flap.
• Figures 158a through 158c are the radiographs of a 38-year-
old man with a previous history of finger and hand infection
treated with amputation 2 years ago. While there has been
no subsequent history of redness or drainage, he now has
difficulty using his hand, especially when grasping objects
such as coins. Examination revealed a gap between his index
and ring fingers, and the index finger rotated toward the
ring. Treatment should consist of?

1. cosmetic prosthetic fitting.


2. functional prosthetic fitting.
3. index metacarpal transposition.
4. metacarpal derotational osteotomy.
5. free toe-to-hand microvascular flap.
Hand Central Ray Resection
• Resection of a central ray indicated for: tumor, infection,
trauma

• Resection of a central ray leads to:


– Inability to grasp (objects fall between fingers)
– Overlap of adjacent fingers, interfering with prehension and
distorting hand symmetry

• Several authors have described bony transposition of the


index ray (figure next slide)
– Muramatsu Arch Orthop Trauma Surg 2007, Hanel J Hand Surg
1993, Posner J Hand Surg 1979

• Neither prosthetics (cosmetic or functional), toe-to-thumb


transfers, nor metacarpal derotational osteotomies have been
described for this indication
2012-163 (Hand)

• Figure 163 is the radiograph of a 68-year-old man with


dorsal radial wrist pain. He rated his pain as 8 on the 0-10
Numeric Pain Rating Scale and said that his pain has
bothered him constantly despite splinting, steroid
injections, and administration of nonsteroidal anti-
inflammatory drugs. Surgical treatment for wrist pain
should consist of?
 
1. scaphoidectomy.
2. radial styloidectomy.
3. proximal row carpectomy.
4. complete wrist arthrodesis.
5. four-corner fusion with scaphoidectomy.
Hand

• Figure 163 is the radiograph of a 68-year-old man with


dorsal radial wrist pain. He rated his pain as 8 on the 0-10
Numeric Pain Rating Scale and said that his pain has
bothered him constantly despite splinting, steroid
injections, and administration of nonsteroidal anti-
inflammatory drugs. Surgical treatment for wrist pain
should consist of?
 
1. scaphoidectomy.
2. radial styloidectomy.
3. proximal row carpectomy.
4. complete wrist arthrodesis.
5. four-corner fusion with scaphoidectomy.
Scapholunate Advanced Collapse (SLAC)

• Injury to SL ligamentpalmar rotatory subluxation


of scaphoidjoint incongruency radiocarpal
arthrosiscapitolunate arthrosis
• Radiolunate typically spared

• This xray demonstrates a typical SLAC wrist pattern


with relative preservation of the radiolunate joint.
He failed conservative treatment options

• Question is: what is the first line treatment option


for SLAC wrist?
Watson Stages

Stage I Arthritis between scaphoid and radial styloid 


Stage II Arthritis between scaphoid and entire scaphoid
facet of the radius 
Stage III  Arthritis between capitate and lunate 

note: radiolunate joint spared 


Scapholunate Advanced Collapse

• CORRECT (Answer 5): scaphoidectomy and four corner


fusion is indicated in Watson II/III SLAC wrists

• Answer 1: insufficient in isolation, need fusion of capitate-


lunate-hamate-triquetrum for stability
• Answer 2: radial styloidectomy is indicated only for Watson
1 SLAC wrists. Would not help in this patient with severe
radiocarpal and capitolunate DJD
• Answer 3: PRC involves excision of scaphoid, lunate,
triquetrum; is contra-indicated in Watson III wrists (this pt)
because the capitate must articulate with the lunate fossa
• Answer 4: is not optimal since there is no motion left in the
wrist; indicated for pancarpal arthritis
2012-182 (Hand)

• In zone II flexor tendon lacerations, repairing only 1 slip compared to


repairing both slips of the flexor digitorum sublimis results in?

• 1. a higher rupture rate.


• 2. profundus bowstringing.
• 3. improved tendon gliding.
• 4. improved passive range of motion.
• 5. proximal interphalangeal joint hyperextension.
• Question 182
• In zone II flexor tendon lacerations, repairing only 1 slip compared to
repairing both slips of the flexor digitorum sublimis results in?

• 1. a higher rupture rate.


• 2. profundus bowstringing.
• 3. improved tendon gliding.
• 4. improved passive range of motion.
• 5. proximal interphalangeal joint hyperextension.
• Zone II (no man’s land) starts at the distal palmar crease and ends at the flexor
digitorum superficialis insertion.
• Zhao, et al, found that gliding resistance was increased with fully intact FDS
compared to FDS with only one slip intact.

• Tan V, Katolik LI. Hand and wrist trauma. In: Flynn JM, ed. Orthopaedic Knowledge Update 10. Rosemont, IL: American Academy of Orthopaedic
Surgeons; 2011:351-362.
• Lutsky K, Boyer M. Flexor tendon injury. In: Trumble TE, Budoff JE, eds. Hand Surgery Update IV. Rosemont, IL: American Society for Surgery of
the Hand; 2007:343-358-511.
• Zhao C, Amadio PC, Zobitz ME, An KN. Resection of the flexor digitorum superficialis reduces gliding resistance in Zone II flexor digitorum profundus
repair in vitro. J Hand Surg. 2002, Mar. PMID: 11901391
2012-210 (Hand)
A 45-year-old man has intermittent elbow pain and
numbness in the fourth and fifth fingers of his left
hand when his elbow is flexed for more than a few
minutes. Past medical history is noncontributory and
he has no known acute injury. The altered sensation is
most likely attributable to?
 
1. axonal degeneration.
2. loss of endoneural tube continuity.
3. displacement of the nodes of Ranvier.
4. mechanical disruption of the perineurium.
5. vascular obstruction of the intraneural vessels.
Question 210
A 45-year-old man has intermittent elbow pain and
numbness in the fourth and fifth fingers of his left
hand when his elbow is flexed for more than a few
minutes. Past medical history is noncontributory and
he has no known acute injury. The altered sensation is
most likely attributable to?
 
1. axonal degeneration.
2. loss of endoneural tube continuity.
3. displacement of the nodes of Ranvier.
4. mechanical disruption of the perineurium.
5. vascular obstruction of the intraneural vessels.
• Paresthesias result from early microvascular
compression and neural ischemia
• Intraneural edema increases over time and
worsens microvascular compression
• Nerve dysfunction can be induced with
extraneural pressure of 4.0 kaP less than
diastolic pressure

Elhassan B, Steinmann SP. Entrapment neuropathy of the ulnar nerve. J Am Acad Orthop Surg. 2007
Nov;15(11):672-81. Review. PubMed PMID: 17989418.
Gupta R, Mozaffar T. Neuromuscular Disease. In: Einhorn TA, O’Keefe RJ, Buckwalter JA, eds.
Orthopaedic Basic Science: Foundations of Clinical Practice. 3rd ed. Rosemont, IL: American Academy
of Orthopaedic Surgeons; 2007:427-443.
2012-213 (Hand)
Figure 213 is the clinical photograph of a 70-year-old woman
with squamous cell cancer on her thumb. Resection and
reconstruction is planned and requires soft-tissue coverage.
Thumb region coverage is best obtained with
 
1. the Moberg flap.
2. a third dorsal metacarpal artery flap.
3. a first dorsal metacarpal artery flap.
4. a full-thickness skin grafting.
5. a reverse cross-finger flap from the index finger with full-
thickness skin grafting.
Image 213
Question 213
Figure 213 is the clinical photograph of a 70-year-old woman
with squamous cell cancer on her thumb.Resection and
reconstruction is planned and requires soft-tissue coverage.
Thumb region coverage is best obtained with
 
1. the Moberg flap.
2. a third dorsal metacarpal artery flap.
3. a first dorsal metacarpal artery flap.
4. a full-thickness skin grafting.
5. a reverse cross-finger flap from the index finger with full-
thickness skin grafting.
Explanation

• First dorsal metacarpal artery (FDMC) flap


– Island pedicle flap off 1st DMCA
– Dorsal thumb soft tissue defects
– Volar thumb soft tissue defects >2cm
– Moberg flap is for volar thumb defects
– 3rd metacarpal artery is less reliably present
– Reverse cross finger for dorsal finger and MP
lesions

Sherif MM. First dorsal metacarpal artery flap in hand reconstruction. I. Anatomical study. J Hand Surg
Am. 1994 Jan;19(1):26-31. PubMed PMID: 8169365.
Sherif MM. First dorsal metacarpal artery flap in hand reconstruction. II. Clinical application. J Hand Surg Am. 1994 Jan;19(1):32-8. PubMed PMID: 8
2012-235 (Hand)

University of Pennsylvania Department of Orthopaedic Surgery


Question 235

University of Pennsylvania Department of Orthopaedic Surgery


Question 235

Dorsal and volar radioulnar ligaments originate at the


sigmoid notch of the radius and insert at the base of the
ulnar styloid

radiocarpal ligament (aka dorsal radiotriquetral ligament)


originates at the radius and inserts on the triquetrum

Dorsal intercarpal ligment runs from the distal aspect of the


scapoid to the medial aspect of the triquetrum

Scapholunate ligament is the primary stabilizer of this


articulation
Answer = 5

University of Pennsylvania Department of Orthopaedic Surgery


2012-245 (Hand)
During the preoperative evaluation of a man with
Dupuytren’s disease who is about to undergo partial
fasciectomy, it is noted that he has a contracture at the
metacarpophalangeal joint level with a pit in the
skin denoting a possible `spiral cord.` This cord displaces
the neurovascular bundle in which direction?

1. Dorsal
2. Medial
3. Midline
4. Dorsolateral
5. Midline and volar

University of Pennsylvania Department of Orthopaedic Surgery


Hand
Question 245

During the preoperative evaluation of a man with


Dupuytren’s disease who is about to undergo partial
fasciectomy, it is noted that he has a contracture at the
metacarpophalangeal joint level with a pit in the
skin denoting a possible `spiral cord.` This cord displaces
the neurovascular bundle in which direction?

1. Dorsal
2. Medial
3. Midline
4. Dorsolateral
5. Midline and volar

University of Pennsylvania Department of Orthopaedic Surgery


Hand
Question 245

Explanation
Dupuytren’s contracture is a fibroproliferative disorder of
autosomal dominant inheritance that most commonly affects
men over age 60 who are of Scandinavian, Irish, or eastern
European descent. Local microvessel ischemia in the hand
and specific platelet-derived and fibroblast growth factors act
at the cellular level to promote the dense myofibroblast
population and altered collagen profiles seen in the affected
tissue.

Normal fascial structures are referred to as bands and ligaments; diseased tissues
are referred to as nodules and cords. The palmar aponeurosis is typically a
triangular thin sheet of fascial tissue that becomes more discretely organized
distally into “pretendinous bands” that travel toward each digit. Pathologic cord
structures form along the pathways of normal fascial anatomy (e.g., spiral bands
become spiral cords). The neurovascular bundle in an involved digit becomes
predictably and profoundly intertwined with disease tissue. The bundle which
normally travels in a straight line, becomes spiraled around the Dupuytren’s cord
and is drawn to the midline of the digits.
1. Benson LS, Williams CS, Kahle M. Dupuytren’s contracture. J Am Acad Orthop Surg. 1998 Jan-
Feb;6(1):24-35. Review. PubMed PMID: 9692938.

University of Pennsylvania Department of Orthopaedic Surgery


2012-251 (Hand)

An infant was born with complex syndactyly involving all 4 fingers of


both hands, short and deformed thumbs, and similar syndactyly
involving both feet. In addition, an altered facial appearance was
noted with protruding eyes, a towered cranium, and midface
hypoplasia. This appearance is characteristic of which syndrome?

1. Apert
2. Poland
3. Holt-Oram
4. VACTERRL
5. Thrombocytopenia-absent radius (TAR)
Answer

• Question 251
An infant was born with complex syndactyly involving all 4 fingers of
both hands, short and deformed thumbs, and similar syndactyly
involving both feet. In addition, an altered facial appearance was
noted with protruding eyes, a towered cranium, and midface
hypoplasia. This appearance is characteristic of which syndrome?

1. Apert
2. Poland
3. Holt-Oram
4. VACTERRL
5. Thrombocytopenia-absent radius (TAR)
Explanation
Apert's syndrome is a congenital disorder causing deformity of the skull, face, hands, and feet. Early fusion of the cranial
and facial suture lines (craniofacial synostosis) results in a variety of skull and facial deformities. The primary deformity
of the hands and feet is severe bilateral complex syndactyly (rosebud hands), often with fusion of the digits. The index,
middle, and ring fingers are affected most often. Cognitive function may be normal or moderately disabled. The genetic
abnormality is FGFr2.

Poland syndrome: unilateral chest wall hypoplasia due to the absence of the sternocostal head of pectoralis major,
forearm and hand shortening and hypoplasia, absence or shortening of the middle phalanx, simple complete syndactaly.

Holt-Oram syndrome: autosomal dominant condition characterized by cardiac defects and associated with radial
deficiency (radial clubhand)

VACTERL syndrome: vertebral anomalies, anal atresia, cardiac abnormalities, tracheoesophageal fistula, renal agenesis,
and limb defects (associated with radial deficiency or radial clubhand).

Thrombocytopenia-absent radius (TAR): autosomal recessive condition with thrombocytopenia and an absent radius. It
can be distinguished from other sydromes associated with radial deficiency in that the thumb is often present.

•RECOMMENDED READINGS:
Upton J. The Apert hand. In: Gupta A, Kay SPJ, Scheker LR, eds. The Growing Hand: Diagnosis and
Management of the Upper Extremity in Children. New York, NY: Mosby; 2000:345-362.
Goldberg MJ, Bartoshesky LE. Congenital hand anomaly: etiology and associated malformations. Hand Clin. 1985
Aug;1(3):405-15. Review. PubMed PMID: 3007544.
2012-253 (Hand)

A 42-year-old woman has had right wrist pain for 2 years. She tried
splint wear and naproxen and has had 3 steroid injections, each
time experiencing less relief. Examination revealed tenderness at
and just
proximal to the radial styloid, with pain exacerbated with thumb
flexion and wrist ulnar deviation. What is the best next step in
treatment?

1. Physical therapy
2. Continued splint wear
3. Repeat injection into the first dorsal wrist compartment
4. Incision of the first dorsal wrist compartment at the volar edge
5. Incision of the first dorsal wrist compartment at the dorsal edge
Answer

• Question 253
A 42-year-old woman has had right wrist pain for 2 years. She tried
splint wear and naproxen and has had 3 steroid injections, each
time experiencing less relief. Examination revealed tenderness at
and just
proximal to the radial styloid, with pain exacerbated with thumb
flexion and wrist ulnar deviation. What is the best next step in
treatment?

1. Physical therapy
2. Continued splint wear
3. Repeat injection into the first dorsal wrist compartment
4. Incision of the first dorsal wrist compartment at the volar edge
5. Incision of the first dorsal wrist compartment at the dorsal
edge
Explanation
De Quervain’s tenosynovitis is a pathologic process of the 1st dorsal (extensor) compartment which
contains the extensor pollicis brevis and abductor pollicis longus tendons. Finkelstein provocative
maneuver is performed with ulnar deviation of the wrist with the thumb clenched in their fist. Pain is
experienced over the 1st dorsal compartment at the level of the radial styloid. This patient has
exhausted nonsurgical management of rest, NSAIDs, thumb spica splinting, and steroid injections x 3,
(not answers 1,2, or 3). The surgical approach is with a transverse incision with the release occurring
on the dorsal side of the 1st compartment to prevent volar subluxation.

•RECOMMENDED READINGS:
Alegado RB, Meals RA. An unusual complication following surgical treatment of deQuervain’s disease. J
Hand Surg Am. 1979 Mar;4(2):185-6. PubMed PMID: 217905.Ashurst JV, Turco DA, Lieb BE.
Tenosynovitis caused by ing: an emerging disease. J Am Osteopath Assoc. 2010 May;110(5):294-6.
PubMed PMID: 20538752.

Ilyas AM, Ast M, Schaffer AA, Thoder J. De quervain tenosynovitis of the wrist. J Am Acad Orthop Surg.
2007 Dec;15(12):757-64.Review. Erratum in: J Am Acad Orthop Surg. 2008 Feb;16(2):35A. Ilyas, Asif
[corrected to Ilyas, Asif M]. PubMed PMID: 18063716.
2012-262 (Hand)

Figure 262 is the video of a 20-year-old man who sustained a striking


injury to the right middle finger metacarpophalangeal joint 1 week ago.
Initial treatment should consist of?
 
1. observation.
2. an immediate return to sports.
3. a metacarpophalangeal joint extension brace.
4. tendon transfer of the extensor indicis proprius.
5. acute repair of the extensor tendon sagittal hood.

University of Pennsylvania Department of Orthopaedic Surgery


Question 262 - Imaging

University of Pennsylvania Department of Orthopaedic Surgery


Question 262 – Preferred Response

Figure 262 is the video of a 20-year-old man who sustained a striking


injury to the right middle finger metacarpophalangeal joint 1 week ago.
Initial treatment should consist of?
 
1. observation.
2. an immediate return to sports.
3. a metacarpophalangeal joint extension brace.
4. tendon transfer of the extensor indicis proprius.
5. acute repair of the extensor tendon sagittal hood.

University of Pennsylvania Department of Orthopaedic Surgery


Question 262 - Explanation

Although we do not have the video here, you may recall from the exam last year that this
video demonstrated subluxation/dislocation of (seen as lateral translation of the extensor
tendon) through a range of motion. In the paper cited below Catalano et al. outline a
technique in which a thermally molded plastic splint is used to hold the MCP joint in 25-35
degrees of hyperextension relative to the adjacent MCP joints. Results were varied but
only one patient eventually required sagittal band reconstruction. The conclusion of the
study is that treatment with a splint was a reasonable 1 st step/treatment.

RECOMMENDED READINGS:
Catalano LW III, Gupta S, Ragland R III, Glickel SZ, Johnson C, Barron OA. Closed treatment of
nonrheumatoid extensor tendon dislocations at the metacarpophalangeal joint. J Hand Surg Am. 2006
Feb;31(2):242-5. PubMed PMID: 16473685.
Lattanza LL, Lam J. Extensor tendon injuries. In: Trumble TE, Budoff JE, eds. Hand Surgery Update IV.
Rosemont, IL: American Society for Surgery of the Hand; 2007:359-370.
 

University of Pennsylvania Department of Orthopaedic Surgery


2012-274 (Hand)

• A 37-year-old woman has a 2-month history of weakness in


thumb and finger extension, but has normal radial deviation
during extension of the wrist. An MRI scan of her forearm
shows no abnormality. She does not recall any traumatic
event. Needle electromyography findings show fibrillations and
reduced recruitment in the extensor pollicis longus, abductor
pollicis longus, extensor digitorum communis, and extensor
carpi ulnaris muscles. Which nerve is most likely compressed?

1. Median
2. Radial
3. Anterior interosseous
4. Posterior interosseous
5. Lateral antebrachial cutaneous
2012-274

• A 37-year-old woman has a 2-month history of weakness in


thumb and finger extension, but has normal radial deviation
during extension of the wrist. An MRI scan of her forearm
shows no abnormality. She does not recall any traumatic
event. Needle electromyography findings show fibrillations and
reduced recruitment in the extensor pollicis longus, abductor
pollicis longus, extensor digitorum communis, and extensor
carpi ulnaris muscles. Which nerve is most likely compressed?

1. Median
2. Radial
3. Anterior interosseous
4. Posterior interosseous
5. Lateral antebrachial cutaneous
2012-274

• Straightforward anatomy question (money in the bank)


• Median:
– Pure motor (no sensory) to wrist flexors
– Hand: 1st and 2nd lumbricals, thenar muscles
• Radial:
– Motor: wrist extensors (except ECU (PIN)), triceps, anconeus, mobile wad
(brachioradialis, ECRL, ECRB)
– Sensory to dorsum of forearm and hand
• AIN:
– Pure motor (no sensory) to FDP, FPL, pronator quadratus
• PIN:
– Motor: hand/finger extensors, ECU, supinator
– Sensory to dorsal wrist capsule; no cutaneous innervation
• Lateral antebrachial cutaneous:
– Pure sensory (no motor) to radial forearm
Question 1

A 33-year-old woman has left index fingertip pain


that is severely exacerbated by reaching
movements. An intense T2 signal under the nailbed
is visible on the MRI scan seen in Figure 1.
•What is the best treatment option?
1. Tumor excision
2. Sympathetic digital block
3. Oral calcium channel blockers
4. Tuft amputation with nail ablation
5. Activity modification and glove wear
Nazerani et al. 2010
Question 1

• A 33-year-old woman has left index fingertip pain


that is severely exacerbated by reaching
movements. An intense T2 signal under the
nailbed is visible on the MRI scan seen in Figure
1.
• What is the best treatment option?
1. Tumor excision
2. Sympathetic digital block
3. Oral calcium channel blockers
4. Tuft amputation with nail ablation
5. Activity modification and glove wear
Glomus Tumors
•Hamartomas of soft tissue of hand. Usually solitary and subungual, and mostly
benign with possible malignant degeneration. They are caused by hyperplasia of
glomus body (vascular complex with rich nerve supply involved in blood flow and
temperature regulation in the dermis, primarily in digits). Pain results from
contractions of glomus cells. Clinical sign is severe pain with localized
tenderness, and cold sensitivity. Surgical excision is best treatment option.

Nazerani S, Motamedi MH, Keramati MR. Diagnosis and management of glomus tumors of the hand. Tech
DavidLNelson.md
Hand Up Extrem Surg. 2010 Mar;14(1):8-13. PubMed PMID: 20216046.
Paliogiannis P, Trignano E, Trignano M. Surgical management of the glomus tumors of the fingers: a single center
experience. Ann Ital Chir. 2011 Nov-Dec;82(6):465-8. PubMed PMID: 22229235.
Wang P, Zhou Z. The treatment of finger glomus tumors by raising a full thickness nail bed flap or finger pulp flap. J
Hand Surg Eur Vol. 2011 Jun;36(5):420-2. PubMed PMID: 21685131.
Question 9

Which treatment technique for segmental nerve defects leads to the


best motor recovery?

1. Silicon tube
2. Collagen tube
3. Nerve allograft
4. Nerve autograft
5. Direct repair with tension
Question 9

Which treatment technique for segmental nerve defects leads to the


best motor recovery?

1. Silicon tube
2. Collagen tube
3. Nerve allograft
4. Nerve autograft
5. Direct repair with tension
Question 9

Nerve autograft is the benchmark for motor nerve defects, sensory


nerve gaps of ≥ 3cm, and mixed defects. For small sensory gaps
where direct tensionless repair is not possible, biologic or synthetic
nerve conduits can be used to bridge sensory nerve gaps of ≤3 cm.

Giusti G, Willems WF, Kremer T, Friedrich PF, Bishop AT, Shin AY. Return of motor
function after segmental nerve loss in a rat model: comparison of autogenous nerve
graft, collagen conduit, and processed allograft (AxoGen). J Bone Joint Surg Am. 2012
Mar 7;94(5):410-7. PubMed PMID: 22398734.

Deal DN, Griffin JW, Hogan MV. Nerve conduits for nerve repair or reconstruction. J Am
Acad Orthop Surg. 2012 Feb;20(2):63-8. Review. PubMed PMID: 22302443.
Question 15

A 24-year-old man with weakness and atrophy of the thumb for 12 months
has very slight numbness on the radial side of his thumb that is constant and
not progressing. He has no other hand or finger numbness. His 2-point static
sensory examination is unremarkable in all digits and there is marked atrophy
of the thenar muscles. His carpal tunnel provocative tests are negative. He
has no symptoms on the opposite hand and otherwise is in excellent health.
Which next step will most likely reveal the diagnosis?
•1. An MRI scan
•2. Muscle biopsy
•3. Carpal tunnel diagnostic injection
•4. Electrodiagnostic testing
•5. Carpal tunnel view radiograph
Question 15

A 24-year-old man with weakness and atrophy of the thumb for 12 months
has very slight numbness on the radial side of his thumb that is constant and
not progressing. He has no other hand or finger numbness. His 2-point static
sensory examination is unremarkable in all digits and there is marked atrophy
of the thenar muscles. His carpal tunnel provocative tests are negative. He
has no symptoms on the opposite hand and otherwise is in excellent health.
Which next step will most likely reveal the diagnosis?
•1. An MRI scan
•2. Muscle biopsy
•3. Carpal tunnel diagnostic injection
•4. Electrodiagnostic testing
•5. Carpal tunnel view radiograph
Question 15
Idiopathic carpal tunnel more likely in middle aged females with bilateral involvement. In
patients with atypical involvement and short period leading up to thenar atrophy, suspect a
space occupying lesion.

EMG unlikely to be helpful because thenar atrophy already seen so likely there will be changes in
velocity on testing. Radiographs won’t show soft tissue mass. Soft tissue injection and
muscle biopsy don’t play a role in discovery of these space occupying lesions.

• Shimizu A, Ikeda M, Kobayashi Y, Saito I, Oka Y. Carpal tunnel syndrome caused by a


ganglion in the carpal tunnel with an atypical type of palsy: a case report. Hand Surg.
2011;16(3):339-41. PubMed PMID: 22072471.
• Chen CH, Wu T, Sun JS, Lin WH, Chen CY. Unusual causes of carpal tunnel syndrome: space
occupying lesions. J Hand Surg Eur Vol. 2012 Jan;37(1):14-9. Epub 2011 Aug 8. PubMed
PMID: 21825010.
• Mauermann ML, Amrami KK, Kuntz NL, Spinner RJ, Dyck PJ, Bosch EP, Engelstad J, Felmlee
JP, Dyck PJ. Longitudinal study of intraneural perineurioma--a benign, focal hypertrophic
neuropathy of youth. Brain. 2009 Aug;132(Pt 8):2265-76. Epub 2009 Jun 30. PubMed PMID:
19567701.
Hand Question 28

Figures 28a and 28b are the pre- and postreduction


radiographs of a finger. The rehabilitation protocol
indicated is?

1)Buddy tape and active motion


2)Static splint in full extension
3)Static splint in 45 degrees of flexion at the
proximal interphalangeal joint
4)Static splint in metacarpophalangeal joint flexion
and proximal interphalangeal joint extension
5)Extension block splint in 90 degrees of flexion at
the proximal interphalangeal joint
Hand Question 28
Hand Question 28

Figures 28a and 28b are the pre- and postreduction


radigraphs of a finger. The rehabilitation protocol
indicated is?

1)Buddy tape and active motion


2)Static splint in full extension
3)Static splint in 45 degrees of flexion at the
proximal interphalangeal joint
4)Static splint in metacarpophalangeal joint flexion
and proximal interphalangeal joint extension
5)Extension block splint in 90 degrees of flexion at
the proximal interphalangeal joint
Hand Question 28

• Treatment of PIP joint injuries is dictated by


stability, with stable injuries treated
symptomatically with 6 weeks of buddy taping
and unstable injuries by dorsal extension block
splinting and incremental extension of splint.
• Morgan WJ, Slowman LS. Acute Hand and Wrist
Injuries in Athletes: Evaluation and Management.
J Am Acad Orthop Surg. 2001;9:389-400
Hand

37

A 27-year-old jackhammer operator has a 4-month history of hand


coldness and severe ischemia that spares his thumb and index finger.
Systemic illnesses have been ruled out. Doppler workup reveals
aneurysmal changes, and digital subtraction arteriogram confirms the
findings. Intervention should consist of
1. Excision and vein graft
2. Surgical thrombectomy
3. Systemic anticoagulatioin
4. intravascular fibrinolysis
5. Interventional embolectomy
Hand

37

A 27-year-old jackhammer operator has a 4-month history of hand


coldness and severe ischemia that spares his thumb and index finger.
Systemic illnesses have been ruled out. Doppler workup reveals
aneurysmal changes, and digital subtraction arteriogram confirms the
findings. Intervention should consist of
1. Excision and vein graft
2. Surgical thrombectomy
3. Systemic anticoagulatioin
4. intravascular fibrinolysis
5. Interventional embolectomy
Based on symptoms what vessels are involved
•Palmar branch of ulnar artery
What’s going on here?
•Hypothenar hammer syndrome
What the heck is that?
•Uncommon! The hook of the hamate strikes the superficial
palmar branch of the ulnar artery in the Guyon space
•Repetitive trauma causes formation of aneurysm and/or
thrombosis of ulnar artery (remember Virchow’s triad!?)
How is it diagnosed?
•Doppler, MRI, CTA, **angiography is gold standard
Treatment algorithm complicated
•Acute thrombosis (less than 2 weeks) try
thrombolytics
•Chronic + Aneurysm  intraoperative
assessment
In chronically occluded (>2wk) aneurysmal
hypothenar hammer syndrome, vascular repair
is necessary.
Yuen JC, Wright E, Johnson LA, Culp WC. Hypothenar hammer syndrome: an update with algorithms
for diagnosis and treatment. Ann Plast Surg. 2011 Oct;67(4):429-38. Review. PubMed PMID:
21372671.
Lifchez SD, Higgins JP. Long-term results of surgical treatment for hypothenar hammer syndrome.
Plast Reconstr Surg. 2009 Jul;124(1):210-6. PubMed PMID: 19568083.
Nitecki S, Anekstein Y, Karram T, Peer A, Bass A. Hypothenar hammer syndrome: apropos of six
cases and review of the literature. Vascular. 2008 Sep-Oct;16(5):279-82. PubMed PMID: 19238870.
Question 48

A mechanic sustained a high-pressure injection of cleaning solvent

into the tip of his index finger 2 hours ago. The finger has good

capillary refill and his 2-point discrimination is 7 mm. Initial treatment

should include

1. a corticosteroid injection.

2. elevation and observation.

3. elective surgical treatment within 7 days.

4. oral clindamycin for 10 days.

5. emergent surgical debridement.

University of Pennsylvania Department of Orthopaedic Surgery


Question 48

A mechanic sustained a high-pressure injection of cleaning solvent into

the tip of his index finger 2 hours ago. The finger has good capillary refill

and his 2-point discrimination is 7 mm. Initial treatment should include

1. a corticosteroid injection.

2. elevation and observation.

3. elective surgical treatment within 7 days.

4. oral clindamycin for 10 days.

5. emergent surgical debridement.

University of Pennsylvania Department of Orthopaedic Surgery


• High pressure injection injuries may lead to amputation in 30% of patients
• Injection site is often innocuous and so frequently there is misdiagnosis and delayed
treatment
• Most often involves nondominant index finger
• Obtain radiographs to look for air or fat planes
• Injury caused by physical trauma, the substance injected, the amount of substance
injected
• Organic solvents are most caustic, causing vasospasm and tissue necrosis
• Less severe injuries (injections with water or air, other non-caustic substances) can be
treated nonsurgically: IV abx, elevation, observation, early motion
• Injection with caustic agents is a surgical emergency
• Surgical debridement <6 hours after initial injury
• Should not use additional solvent to clean organic compound injections

High pressure injection with a caustic agent is a surgical emergency which should
be evaluated with radiographs and taken for debridement regardless of the
innocuous appearance of the injury or absence of symptoms.

RECOMMENDED READINGS

Tan V, Katolik LI. Hand and wrist trauma. In; Flynn JM, ed. Orthopaedic Knowledge Update 10. Rosemont, IL: American Academy of Orthopaedic

Surgeons; 2011:351-362.

Stevanovic MV, Sharpe F. Acute infections. In: Wolfe SW, Hotchkiss RN, Pederson WC, Kozin SH, eds. Green’s Operative Hand Surgery. 6th ed.

Philadelphia, PA: Elsevier Churchill Livingstone; 2011:81.


University of Pennsylvania Department of Orthopaedic Surgery
Question 58

• An otherwise healthy 42-year-old woman is


scheduled for carpal tunnel release. The
physician should adhere to routine sterility
protocols:

1. without local or systemic antibiotics.


2. and irrigate with cefazolin solution.
3. and irrigate with bacitracin solution.
4. and administer cefazolin within 1 hour before incision.
5. and administer cefazolin within 1 hour before incision and
continue dosing up to 23 hours after surgery.
Question 58

• An otherwise healthy 42-year-old woman is


scheduled for carpal tunnel release. The
physician should adhere to routine sterility
protocols:

1. without local or systemic antibiotics.


2. and irrigate with cefazolin solution.
3. and irrigate with bacitracin solution.
4. and administer cefazolin within 1 hour before incision.
5. and administer cefazolin within 1 hour before incision and
continue dosing up to 23 hours after surgery.
Question 58

Multiple retrospective and prospective studies investigating the


use of perioperative antibiotics in clean, elective hand surgery
(including carpal tunnel releases) found no significant difference
in post-operative infection rates among those who received
perioperative antibiotics and those who did not.

Tosti R, Fowler J, Dwyer J, Maltenfort M, Thoder JJ, Ilyas AM. Is antibiotic prophylaxis
necessary in elective soft tissue hand surgery? Orthopedics. 2012 Jun 1;35(6):e829-33.
doi: 10.3928/01477447-20120525-20. PubMed PMID: 22691653.
Bykowski MR, Sivak WN, Cray J, Buterbaugh G, Imbriglia JE, Lee WP. Assessing the impact
of antibiotic prophylaxis in outpatient elective hand surgery: a single-center, retrospective
review of 8,850 cases. J Hand Surg Am. 2011 Nov;36(11):1741-7. Epub 2011 Oct 5.
PubMed PMID: 21975095.
Harness NG, Inacio MC, Pfeil FF, Paxton LW. Rate of infection after carpal tunnel release
surgery and effect of antibiotic prophylaxis. J Hand Surg Am. 2010 Feb;35(2):189-96.
PubMed PMID: 20141890.
Rizvi M, Bille B, Holtom P, Schnall SB. The role of prophylactic antibiotics in elective hand
surgery. J Hand Surg Am. 2008 Mar;33(3):413-20. Review. PubMed PMID: 18343301.
*Question 65 (Hand)

• Figures 65a and 65b are the magnetic resonance


arthrogram and wrist arthroscopic photograph of a 25-
year-old man who has wrist pain during extension and
ulnar rotation. Treatment should consist of
• 1. synovectomy.
• 2. ulnar shortening osteotomy.
• 3. diagnostic arthroscopy only.
• 4. triangular fibrocartilage complex tear debridement.
• 5. triangular fibrocartilage complex repair dorsal ligament.
*Question 65 (Hand)

• Figures 65a and 65b are the magnetic resonance


arthrogram and wrist arthroscopic photograph of a
• 25-year-old man who has wrist pain during extension and
ulnar rotation. Treatment should consist of
• 1. synovectomy.
• 2. ulnar shortening osteotomy.
• 3. diagnostic arthroscopy only.
• 4. triangular fibrocartilage complex tear debridement.
• 5. triangular fibrocartilage complex repair dorsal ligament.
• Type I TFCC tear (traumatic)
– Extended wrist with forearm pronation
• Type II TFCC tear (degenerative)
– Assoc with ulnar variance, ulnocarpal impaction
• TFCC consists of
– Dorsal and volar radioulnar ligament, Central articular disc, Meniscus
homolog, Ulnar collateral ligament, ECU subsheath, Origin of ulnolunate
and ulnotriqeutral ligament
• Physical exam
– Pain to ulnar deviation (TFCC comp) or radial deviation (TFCC tension)
– “Fovea” sign, tenderness soft spot between ulnar styloid and FCU
• Nonop treatment
– Immobilization, NSAIDS, steroid inj
• Operative treatment
– Arthroscopic debridement for type 1A
– Arthroscopic repair for type 1B,C,D
– Wafer procedure for type 2A-C, where pos ulnar variance <2mm
– Limited ulnar head resection for type 2D
Question 75 Hand

The least gliding resistance for a flexor tendon


laceration at the thumb palmar-digital crease as
shown in Figure 75 can be achieved with
1. a 6-strand repair.
2. a division and repair.
3. debridement of the partial laceration.
4. no debridement, motion therapy only.
5. multiple-strand core repair with epitendinous
repair.
Question 75 Hand
Question 75 Hand

The least gliding resistance for a flexor tendon


laceration at the thumb palmar-digital crease as
shown in Figure 75 can be achieved with
1. a 6-strand repair.
2. a division and repair.
3. debridement of the partial laceration.
4. no debridement, motion therapy only.
5. multiple-strand core repair with epitendinous
repair.
Question 75 Hand
Complications related to partial tendon injuries include triggering,
entrapment, and secondary rupture. Evidence is mixed whether
partial lacerations >50% require repair to reduce triggering or
rupture. If a partial laceration is causing triggering, trimming is
recommended

Partial tendon injuries treating with trimming is an adequate


alternative to repair with respect to both strength and gliding
resistance.

RECOMMENDED READINGS
Lilly SI, Messer TM. Complications after treatment of flexor tendon
injuries. J Am Acad Orthop Surg. 2006 Jul;14(7):387-96. Review.
PubMed PMID: 16822886.
Mostofi A, Palmer J, Akelman E. Flexor tendon injury. In: Chung KC,
Murray PM, eds. Hand Surgery Update 5. Rosemont, IL: American
Society for Surgery of the Hand; 2012:181-192.
Trauma
Question 78

A 42-year-old woman has the injury shown in Figures 78a and 78b. The
decision to treat the ulnar styloid surgically is based upon which finding?

1. Patient age

2. Displacement of the radius fracture

3. Displacement of the ulnar styloid fracture

4. Position of the ulnar styloid after open reduction and internal fixation of the
radius

5. Stability of the distal radioulnar joint after open reduction and internal fixation
of the radius
Imaging

Figure 78a Figure 78b


Question 78

A 42-year-old woman has the injury shown in Figures 78a and 78b. The
decision to treat the ulnar styloid surgically is based upon which finding?

1. Patient age

2. Displacement of the radius fracture

3. Displacement of the ulnar styloid fracture

4. Position of the ulnar styloid after open reduction and internal fixation of the
radius

5. Stability of the distal radioulnar joint after open reduction and internal fixation
of the radius
Question 78 Explained

• Sammer DM, Shah HM, Shauver MJ, Chung KC. The


effect of ulnar styloid fractures on patient-rated
outcomes after volar locking plating of distal radius
fractures. J Hand Surg Am. 2009 Nov;34(9):1595-
602. PubMed PMID: 19896004.
– Because the stabilizing ligaments of the DRUJ insert
on the ulnar styloid base, there are 2 clinically
distinct types of nonunion based on the stability of
the DRUJ. One should always examine the stability
of the DRUJ intraoperatively.
• If there is instability of the DRUJ in patients
with DRF and ulnar styloid fractures, the
ulnar styloid should also be fixed.
Hand

95. The deformity caused by long-term


arthritis of the first carpometacarpal joint
of the hand often leads to a secondary
hyperextension arthrosis of which joint?

1. Midcarpal
2. Radiocarpal
3. Scaphotrapezotrapezoidal
4. Thumb interphalangeal
5. Thumb metacarpophalangeal

University of Pennsylvania Department of Orthopaedic Surgery


University of Pennsylvania Department of Orthopaedic Surgery
Hand

95. The deformity caused by long-term


arthritis of the first carpometacarpal joint
of the hand often leads to a secondary
hyperextension arthrosis of which joint?

1. Midcarpal
2. Radiocarpal
3. Scaphotrapezotrapezoidal
4. Thumb interphalangeal
5. Thumb metacarpophalangeal

University of Pennsylvania Department of Orthopaedic Surgery


Explanation:
Van Heest AE, Kallemeier P. Thumb carpal metacarpal
arthritis. J Am Acad Orthop Surg. 2008. Review.
-1st CMC arthritis: pain over thumb adductors and
thenar muscles
-avoidance of painful thumb abduction leads to
adduction deformity in 1st web space
-continued stiffness/adduction of 1st CMC leads to
thumb MCP hyperextension deformity to
compensate for lost motion

Testable concept: The natural progression of


1st CMC arthritis includes adduction contracture and then
thumb MCP hyperextension deformity to compensate for
lost motion

University of Pennsylvania Department of Orthopaedic Surgery


Hand

103: Figure 103 is the clinical photograph of a 62-year-old man with


numbness and weakness that has been progressing for 10 years. What
is the most appropriate treatment to improve thumb function?

1: Functional splinting

2: Neurotization of the thenar muscles

3: Hypothenar muscle transfer to thumb intrinsic

4: Arthrodesis of the thumb carpometacarpal joint in abduction

5: Transfer of the extensor indicis proprius around the ulnar wrist


103: Figure 103 is the clinical photograph of a 62-year-old
man with numbness and weakness that has been
progressing for 10 years. What is the most appropriate
treatment to improve thumb function?

1: Functional splinting

2: Neurotization of the thenar muscles

3: Hypothenar muscle transfer to thumb intrinsic

4: Arthrodesis of the thumb carpometacarpal joint in


abduction

5: Transfer of the extensor indicis proprius around the


ulnar wrist
Explanation
Thenar muscle wasting is indicative of median nerve
dysfunction. Opponenplasty is performed after long term palsy.
Extensor indicis, ring FDS, ECRL or palmaris longus (Camitiz)
can be used for opponenplasty. Extensor indicis opponenplasty
may be best if FDS tendons previously cut, high median nerve
palsy, or insensate thumb.

- Anderson GA, Lee V, Sundararaj GD. Opponensplasty by extensor indicis and

flexor digitorum superficialis tendon transfer. J Hand Surg Br. 1992

Dec;17(6):611-4. PubMed PMID: 1484241.

Burkhalter W, Christensen RC, Brown P. Extensor indicis proprius

opponensplasty. J Bone Joint Surg Am. 1973 Jun;55(4):725-32. PubMed PMID:

4283744.

- Cawrse NH, Sammut D. A modification in technique of abductor digiti minimi

(Huber) opponensplasty. J Hand Surg Br. 2003 Jun;28(3):233-7. PubMed PMID:

12809655
Question 133

Figures 133a and 133b are the clinical photographs of a


34-year-old woman with increasing pain in her index finger
for 3 days. The pain is worse with passive extension.
Appropriate treatment should consist of

1. observation.
2. hand therapy.
3. oral antibiotics.
4. intravenous antibiotics.
5. irrigation and debridement of the flexor tendon sheath.

University of Pennsylvania Department of Orthopaedic Surgery


Question 133 - Imaging

University of Pennsylvania Department of Orthopaedic Surgery


Question 133 – Preferred Response

Figures 133a and 133b are the clinical photographs of a


34-year-old woman with increasing pain in her index finger
for 3 days. The pain is worse with passive extension.
Appropriate treatment should consist of

1. observation.
2. hand therapy.
3. oral antibiotics.
4. intravenous antibiotics.
5. irrigation and debridement of the flexor tendon sheath.

University of Pennsylvania Department of Orthopaedic Surgery


Question 133 - Explanation

Diagnosis is Flexor Tenosynovitis

Kanavel signs:
1. Fusiform swelling of entire involved digit
2. Exquisite tenderness along flexor tendon
3. Semiflexed resting posture
4. Pain with passive extension of digit

Treatment of flexor tenosynovitis:


If early (<48 hours), may consider IV abx alone
>48 hours, requires surgical debridement of flexor tendon sheath and IV abx

Flexor tenosynovitis is diagnosed clinically using Kanavel signs and hallmark of treatment
is IV antibiotics and surgical debridement for all cases presenting >48 hours following
penetrating trauma.
Capo JT. Infections. In: Chung KC, Murray PM, eds. Hand Surgery Update 5. Rosemont, IL: American
Society for Surgery of the Hand; 2012:394-399.

Draeger RW, Bynum DK Jr. Flexor tendon sheath infections of the hand. J Am Acad Orthop Surg. 2012
Jun;20(6):373-82. Review. PubMed PMID: 22661567.

University of Pennsylvania Department of Orthopaedic Surgery


140

• A 38-year-old woman had a distal radius


fracture treated with a short-arm cast 3
months ago. The fracture healed in good
alignment. Figure 140 shows her attempt to
extend her thumb. What is the best
treatment option?
– Static splinting
– Dynamic splinting
– Transfer of the extensor pollicis brevis
– Transfer of the extensor indicis proprius
– Arthrodesis of the interphalangeal joint
– Static splinting
– Dynamic splinting
– Transfer of the extensor pollicis brevis
– Transfer of the extensor indicis proprius
– Arthrodesis of the interphalangeal joint
• Roth KM, Blazar PE, Earp BE, Han R, Leung A.
Incidence of extensor pollicis longus tendon
rupture after nondisplaced distal radius fractures.
J Hand Surg Am. 2012 May;37(5):942-7. Epub
2012 Mar 29. PubMed PMID: 22463927.

• Testable concept – EPL rupture following DR


fracture
• EPL ruptures most common following undisplaced
DR fracture. Possibly due to pressure on tendon
causing attritional wear.
Question 213

Figures 213a and 213b are the clinical photograph and biopsy
specimen of a 65-year-old man with a lesion under his thumbnail
that was biopsied by a dermatologist. Appropriate treatment
should consist of

1. observation.
2. local excision.
3. marginal excision.
4. thumb ray resection.
5. amputation at the interphalangeal joint.
Question 213 Images
Question 213

Figures 213a and 213b are the clinical photograph and biopsy
specimen of a 65-year-old man with a lesion under his thumbnail
that was biopsied by a dermatologist. Appropriate treatment
should consist of

1. Observation
2. Local excision
3. Marginal excision
4. Thumb ray resection
5. Amputation at the interphalangeal joint
Question 213

• PREFERRED RESPONSE: 5

• Subungual melanomas usually present in advanced stages and the


treatment for a lesion of the thumb is amputation. An amputation
at the IP joint with clear margins does equally as well as a more
proximal amputation. These lesions are most likely to
metastasize to the lung.

• Haase SC, Chung KC. Skin tumors. In: Wolfe SW, Hotchkiss RN,
Pederson WC, Kozin SH, eds. Green’s Operative Hand Surgery.
6th ed. Philadelphia, PA: Elsevier Churchill Livingstone;
2011:2131-2133.
• Plate AM, Steiner G, Posner MA. Malignant tumors of the hand and
wrist. J Am Acad Orthop Surg. 2006 Nov;14(12):680-92. PubMed
PMID: 17077340.
Hand
Question 225

• Figure 225 is the clinical photograph of a 26-year-old man


who fell through a window and sustained a laceration to his
thumb 5 days ago. He is unable to flex his thumb.
Treatment should include

• 1. palmaris longus tendon transfer.


• 2. reconstruction with a palmaris longus free tendon bridge
graft.
• 3. direct repair of the flexor pollicis longus with core
sutures only.
• 4. repair of the flexor pollicis longus with core and
epitendinous sutures.
• 5. transfer of the flexor digitorum superficialis of the ring
finger to the thumb.
Question 225
Question 225

• Figure 225 is the clinical photograph of a 26-year-old man


who fell through a window and sustained a laceration to his
thumb 5 days ago. He is unable to flex his thumb.
Treatment should include

• 1. palmaris longus tendon transfer.


• 2. reconstruction with a palmaris longus free tendon bridge
graft.
• 3. direct repair of the flexor pollicis longus with core
sutures only.
• 4. repair of the flexor pollicis longus with core and
epitendinous sutures.
• 5. transfer of the flexor digitorum superficialis of the ring
finger to the thumb.
Question 225
• The patient has a FPL tendon laceration, as indicated by his
inability to flex his thumb. Since the injury is acute, it is amenable
to direct repair (choices 3 or 4). Studies have shown that
epitendinous sutures improve tendon gliding and strength of repair
(add 20% to tensile strength). Therefore, choice 4 is correct.

• RECOMMENDED READINGS
• Gulihar A, Hajipour L, Dias JJ. Comparison of three different peripheral suturing techniques for
partial flexor tendon lacerations: a controlled in-vitro biomechanical study. Hand Surg. 2012
2012;17(2):155-160. PubMed PMID: 22745077.
• Lee SK, Goldstein RY, Zingman A, Terranova C, Nasser P, Hausman MR. The effects of core
suture purchase on the biomechanical characteristics of a multistrand locking flexor tendon
repair: a cadaveric study. J Hand Surg Am. 2010 Jul;35(7):1165-71. Epub 2010 Jun 11. PubMed
PMID: 20541326.
• Coats RW II, Echevarría-Oré JC, Mass DP. Acute flexor tendon repairs in zone II. Hand Clin. 2005
May;21(2):173-9. Review. PubMed PMID: 15882596.
• Barrie KA, Wolfe SW, Shean C, Shenbagamurthi D, Slade JF III, Panjabi MM. A biomechanical
comparison of multistrand flexor tendon repairs using an in situ testing model. J Hand Surg Am.
2000 May;25(3):499-506. PubMed PMID: 10811755.
230. Figures 230a through 230d are the pre- and
postreduction radiographs of a 6-year-old boy who had a
fracture of the radius and ulna shafts in the distal diaphyses.
Successful reduction of the completely displaced fractures is
achieved. To best maintain reduction while minimizing
complications, treatment should include immobilization in a

1. removable splint.
2. sugar-tong splint.
3. short-arm cast.
4. long-arm cast.
5. long-arm thumb spica cast.

University of Pennsylvania Department of Orthopaedic Surgery


Figure 230a Figure 230b Figure 230c Figure 230d
230. Figures 230a through 230d are the pre- and
postreduction radiographs of a 6-year-old boy who had a
fracture of the radius and ulna shafts in the distal diaphyses.
Successful reduction of the completely displaced fractures is
achieved. To best maintain reduction while minimizing
complications, treatment should include immobilization in a

1. removable splint.
2. sugar-tong splint.
3. short-arm cast.
4. long-arm cast.
5. long-arm thumb spica cast.

University of Pennsylvania Department of Orthopaedic Surgery


3. short-arm cast.

This is contradiction to what we do at CHOP


typically however the study show that below-elbow
cast treatment was comparable in terms of
redisplacement, union time, and movement of
the wrist to the above-elbow treatment. Splints
are good for temporary immobilization and to
allow for swelling but should not be used as
definitive treatment.

University of Pennsylvania Department of Orthopaedic Surgery


Tumor

269

• Figures 269a and 269b are the MRI scans of a 60-year-old man
who has pain and loss of elbow flexion strength. In addition to the
distal biceps tendon injury, what is the most likely diagnosis?
• 1. Soft-tissue sarcoma
• 2. Intraneural ganglion cyst
• 3. Denervation of the biceps muscle
• 4. Benign peripheral nerve sheath tumor
• 5. Malignant peripheral nerve sheath tumor
269
269

• Figures 269a and 269b are the MRI scans of a 60-year-old man
who has pain and loss of elbow flexion strength. In addition to the
distal biceps tendon injury, what is the most likely diagnosis?
• 1. Soft-tissue sarcoma
• 2. Intraneural ganglion cyst
• 3. Denervation of the biceps muscle
• 4. Benign peripheral nerve sheath tumor
• 5. Malignant peripheral nerve sheath tumor
269
Bhargava et al described T2-weighted MR imaging of soft tissue tumors of neural origin
as showing round lesions with a central hypointensity and a hyperintense rim
resembling a target. They defined the "target sign" as a mass consisting of a solitary
target, or a multicompartmental mass in which the largest component consists of
multiple targets.
In their study, this target sign was seen in all 12 neurofibromas and only 1 of the 11
malignant peripheral nerve sheath tumors. Their statistical analysis showed good
differentiation of benign and malignant tumors using this sign (chi = 0.91).
The mass appears to arise from the median nerve.
A ganglion would appear homogenously fluid filled.
A well-circumscribed mass that is found incidentally,
is more likely to be benign than malignant.
Fatty infiltration + atrophy are typical of denervation.

Benign peripheral nerve sheath tumors have a “target”


appearance on MRI
• RECOMMENDED READINGS
• Bhargava R, Parham DM, Lasater OE, Chari RS, Chen G, Fletcher BD. MR imaging differentiation of benign and
malignant peripheral nerve sheath tumors: use of the target sign. Pediatr Radiol. 1997 Feb;27(2):124-9. PubMed PMID:
9028843.
• Uetani M, Hashmi R, Hayashi K, Nagatani Y, Narabayashi Y, Imamura K. Peripheral nerve intraneural ganglion cyst: MR
findings in three cases.
Question 8
A patient has weakness with shoulder abduction and elbow flexion after a
motorcycle collision. During evaluation with electromyography and nerve
conduction studies, what is the significance of a normal signal in the
rhomboid muscle?

•1. Predicts recovery of the biceps


•2. Predicts recovery of the deltoid
•3. Identifies a brachial plexus injury
•4. Identifies an injury to the cervical spinal cord
•5. Identifies a cervical root avulsion from the spinal cord
Question 8
A patient has weakness with shoulder abduction and elbow flexion after a
motorcycle collision. During evaluation with electromyography and nerve
conduction studies, what is the significance of a normal signal in the
rhomboid muscle?

•1. Predicts recovery of the biceps


•2. Predicts recovery of the deltoid
•3. Identifies a brachial plexus injury
•4. Identifies an injury to the cervical spinal cord
•5. Identifies a cervical root avulsion from the spinal cord
Question 8
The dorsal scapular nerve innervates the rhomboids. It originates from the C5 nerve root directly,
immediately proximal to the point where the roots of C5 – T1 merge to form the trunks of the brachial
plexus. As such, if the rhomboids are functional on EMG, it signifies that the injury is distal to the origin
of the dorsal scapular nerve, i.e in the brachial plexus. If the rhomboids were abnormal on EMG,
suspicion for a C5 injury would be high.

RECOMMENDED READINGS
Tubbs RS, Tyler-Kabara EC, Aikens AC, Martin JP, Weed LL, Salter EG, Oakes WJ. Surgical anatomy of the dorsal scapular nerve. J Neurosurg.
2005 May;102(5):910-1. PubMed PMID: 15926718.
Balakrishnan G, Kadadi BK. Clinical examination versus routine and paraspinal electromyographic studies in predicting the site of lesion in
brachial plexus injury. J Hand Surg Am. 2004 Jan;29(1):140-3.
PubMed PMID: 14751117.
Question 15
•A 30-year-old man sustained a C5-C6 nerve root avulsion 3
months ago as a result of a motor vehicle collision. What is the
most appropriate treatment option?
–1. Explore the brachial plexus and repair avulsed nerve roots
–2. Transfer tendon of the posterior deltoid to the central tendon of the
biceps
–3. Transfer the intercostal nerves to the posterior cord of the brachial
plexus
–4. Recommend surgical reconstruction if elbow flexion does not return
within 6 months
–5. Transfer fascicles of the ulnar nerve to the motor nerve of the
biceps and fascicles of the median nerve to the motor nerve of the
brachialis
Question 15
•A 30-year-old man sustained a C5-C6 nerve root avulsion 3
months ago as a result of a motor vehicle collision. What is the
most appropriate treatment option?
–1. Explore the brachial plexus and repair avulsed nerve roots
–2. Transfer tendon of the posterior deltoid to the central tendon of the
biceps
–3. Transfer the intercostal nerves to the posterior cord of the brachial
plexus
–4. Recommend surgical reconstruction if elbow flexion does not return
within 6 months
–5. Transfer fascicles of the ulnar nerve to the motor nerve of the
biceps and fascicles of the median nerve to the motor nerve of the
brachialis
Double nerve transfer to restore elbow flexion in
brachial plexus injury
•Restoration of elbow flexion is the main objective in the treatment of
brachial plexus palsies affecting the upper roots. Transfer of the ulnar
nerve to the nerve of the biceps has given satisfactory results, but a
double nerve transfer: one or more fascicles of the ulnar nerve to the
nerve to the biceps and a fascicle of the median nerve to the motor
branch to the brachialis muscle yields superior results. The percentage of
success and the strength of elbow flexion restored were increased without
any morbidity. 

•Liverneaux PA, Diaz LC, Beaulieu JY, Durand S, Oberlin C. Preliminary results of double nerve transfer to restore elbow
flexion in upper type brachial plexus palsies. Plast Reconstr Surg. 2006 Mar;117(3):915- 9. PubMed PMID: 16525285.
•Teboul F, Kakkar R, Ameur N, Beaulieu JY, Oberlin C. Transfer of fascicles from the ulnar nerve to the nerve to the biceps in
the treatment of upper brachial plexus palsy. J Bone Joint Surg Am. 2004 Jul;86-A(7):1485-90. PubMed PMID: 15252097.
Question 28
A 22-year-old man injured the dorsal aspect of his long finger
with a grinder wheel. The defect over the middle phalanx is 1.5 x
1 cm with the bone exposed and no periosteum. Which
procedure will provide soft-tissue coverage of the defect?

1. Axial flap
2. Full-thickness skin graft
3. Split-thickness skin graft
4. Cross-finger flap from the index finger
5. Reverse cross-finger flap from the ring finger
Question 28
A 22-year-old man injured the dorsal aspect of his long finger
with a grinder wheel. The defect over the middle phalanx is 1.5 x
1 cm with the bone exposed and no periosteum. Which
procedure will provide soft-tissue coverage of the defect?

1. Axial flap
2. Full-thickness skin graft
3. Split-thickness skin graft
4. Cross-finger flap from the index finger
5. Reverse cross-finger flap from the ring finger
Question 28
• The cross-finger subcutaneous flap has been successfully used
to reconstruct the avulsed eponychial skin fold and cover
large nail bed defects with exposed bone denuded of
periosteum, especially of the dorsal finger
• Full thickness skin grafts are used to cover the donor and
recipient site
• Cross finger flap is used for volar finger injury

Atasoy E. Reversed cross-finger


subcutaneous flap. J Hand Surg Am.
1982 Sep;7(5):481-3.
Question 39
•A 2-year-old girl has a 1-year history of atraumatic stiffness
of her middle finger. Her mother observes frequent locking of
the finger into flexion. This condition is most successfully
treated with
•1. nighttime static splinting into extension.
•2. surgical release of the first annular (A-1) pulley.
•3. excision of 1 slip of the sublimus flexor digitorum
superficialis tendon.
•4. corticosteroid injection of the flexor tendon sheath.
•5. observation with expected spontaneous resolution.
Question 39
•A 2-year-old girl has a 1-year history of atraumatic stiffness
of her middle finger. Her mother observes frequent locking of
the finger into flexion. This condition is most successfully
treated with
•1. nighttime static splinting into extension.
•2. surgical release of the first annular (A-1) pulley.
•3. excision of 1 slip of the sublimus flexor digitorum
superficialis tendon.
•4. corticosteroid injection of the flexor tendon sheath.
•5. observation with expected spontaneous resolution.
Trigger finger
•In children, trigger finger are different from trigger thumbs. It is less common than in adults and have variable causes. As such, an A1 pulley release alone will not
always correct the triggering. Additional treatments, such as resection of one or both limbs of sublimis tendon or an A3 pulley release may be required. In this case,
the child has had symptoms for over a year and is unlikely to respond to nonop tx.

•RECOMMENDED READINGS
•Moon WN, Suh SW, Kim IC. Trigger digits in children. J Hand Surg Br. 2001 Feb;26(1):11-2. PubMed
•PMID: 11162006.
•Cardon LJ, Ezaki M, Carter PR. Trigger finger in children. J Hand Surg Am. 1999 Nov;24(6):1156-61.
•PubMed PMID: 10584935.
Question 65
•Following hand tendon suture (repair or transfer),
which method of intrasurgical inspection provides
the best understanding of translation, gliding, and
tension?
•1. Tenodesis examination
•2. Active motion during surgery
•3. Passive extension of the wrist and digit
•4. Nerve stimulation to generate finger motion
•5. Observation for kinking of the tendon during
passive flexion and extension
Question 65
•Following hand tendon suture (repair or transfer),
which method of intrasurgical inspection provides
the best understanding of translation, gliding, and
tension?
•1. Tenodesis examination
•2. Active motion during surgery
•3. Passive extension of the wrist and digit
•4. Nerve stimulation to generate finger motion
•5. Observation for kinking of the tendon during
passive flexion and extension
Question 65 Explanation
•Wide-awake tendon repair performed under local anesthesia using
lidocaine with epinephrine without a tourniquet. It allows for
intraoperative assessment for repair gaps by having the awake
patient to actively flex or extend the digit.
•It reduces the need for postop tenolysis by allowing intraoperative
assessment of whether the repair will fit through pulleys. It allows
for on-the-spot debulking of bunched repairs and allows division of
A4 pulley and venting (partial division) of A2 pulleys.
•It also facilitates postop early active motion
Question 75
Question 75

• Hypothenar Hammer Syndrome: Ulnar artery aneurism caused by repetitive motion about
the wrist (manual laborers)
• Ulnar artery is susceptible to repetitive stress as it passes through Guyon’s Canal around
the hook of hamate
• Treatment is aneurism excision with reconstruction (vein > artery)
• Distal signs/symptoms are caused by emboli from the aneurism, necessitating excision.
Question 75
Question 96
Contracture of the web space in Dupuytren’s disease is
attributed to involvement of which structure?
1. Spiral band
2. Natatory ligament
3. Grayson’s ligament
4. Pretendinous bands
5. Septa of Legueu and Juvara
Question 96
Contracture of the web space in Dupuytren’s disease is
attributed to involvement of which structure?
1. Spiral band
2. Natatory ligament
3. Grayson’s ligament
4. Pretendinous bands
5. Septa of Legueu and Juvara
Recommended Reading
Natatory Ligament – causes web space contraction

Spiral band is the most clinically significant


-causes PIP contraction
-travels underneath the neurovasc. bundle – proliferation
displaces the neurovascular bundle centrally and very
superficial (risk of damage in surgery)
-inserts onto Grayson’s ligament

Central cord – causes MCP contracture

Retrovascular cord – causes DIP contracture

Cleland’s ligament = only ligament NOT involved in dupuytren’s

Septa of Lequeu and Juvara – typically not involved

•Strickland JW, Leibovic SJ. Anatomy and pathogenesis of the digital cords and nodules. Hand Clin. 1991 Nov;7(4):645-57;
discussion 659-60. Review. PubMed PMID: 1769987.
•McFarlane RM. The anatomy of Dupuytren’s disease. Bull Hosp Jt Dis Orthop Inst. 1984 Fall;44(2):318-37. PubMed PMID:
6099177.
Question 103
What is a relative contraindication to replantation in the hand?

1.Presence of an arterial “ribbon sign”


2.Amputation of a single nonborder digit
3.Cold ischemia time of 12 hours in an amputated digit
4.Multiple digit amputations in the nondominant hand
5.Amputation of a digit distal to the flexor digitorum sublimus
tendon
Question 103
What is a relative contraindication to replantation in the hand?

1.Presence of an arterial “ribbon sign”


2.Amputation of a single nonborder digit
3.Cold ischemia time of 12 hours in an amputated digit
4.Multiple digit amputations in the nondominant hand
5.Amputation of a digit distal to the flexor digitorum sublimus
tendon
Arterial Ribbon Sign
A marked tortuosity of the digital arteries ("ribbon" sign) at the site of a digit amputation
is a sign of a severe avulsion injury to the neurovascular bundles. Predictive signs of
severe damage to the neurovascular bundle and unsuccessful replantation include the
“red line” and “ribbon” signs, which suggest a wide zone of intimal injury

RECOMMENDED READINGS
Friedrich JB, Poppler LH, Mack CD, Rivara FP, Levin LS, Klein MB. Epidemiology of upper extremity replantation surgery in the United States. J Hand
Surg Am. 2011 Nov;36(11):1835-40. doi: 10.1016/j. jhsa.2011.08.002. Epub 2011 Oct 5. PubMed PMID: 21975098.
Soucacos PN. Indications and selection for digital amputation and replantation. J Hand Surg Br. 2001 Dec;26(6):572-81. Review. PubMed PMID:
11884116.
Question 133
Which clinical feature is most likely associated with the radiograph shown in
Figure 133?

1. Skin thickening and hardening


2. Sacroiliac joint pain and stiffness
3. Activity-related pain relieved by rest
4. Erythematous plaques and pruritic scaling
5. Pain and swelling precipitated by excessive alcohol use
Question 133
Which clinical feature is most likely associated with the radiograph shown in
Figure 133?

1. Skin thickening and hardening


2. Sacroiliac joint pain and stiffness
3. Activity-related pain relieved by rest
4. Erythematous plaques and pruritic scaling
5. Pain and swelling precipitated by excessive alcohol use
Psoriatic arthritis in the hand

Seronegative spondyloarthropathy that presents with asymmetiric oligo/monoarticular arthritis


affecting DIP(predominant), PIP, MCP, dactylitis, nail pitting, lifting of the nail plate, and arthritis
mutilans. Affects 5-20% of patients with psoriasis. HLAB27 positive in 50%. DIP “pencil-in-cup”
deformity on XR. Treatment options include NSAIDs, methotrexate, sulfasalazine, cyclosporine,
TNF-alpha inhibitors. Surgical options include digit fusion vs resection arthroplasty.

Rose JH, Belsky MR. Psoriatic arthritis in the hand. Hand Clin. 1989 May;5(2):137-44. Review.
PubMed PMID: 2661570.
Day MS, Nam D, Goodman S, Su EP, Figgie M. Psoriatic arthritis. J Am Acad Orthop Surg. 2012
Jan;20(1):28-37. doi: 10.5435/JAAOS-20-01-028. Review. PubMed PMID: 22207516.
Question 140
A 40-year-old woman has no active thumb flexion after 2 attempts at repair of her zone II flexor pollicis longus (FPL) laceration during the last 4 months. Extension is also
limited with her wrist and thumb extended, but there is improved interphalangeal extension with the MCP joint flexed. She would like to maintain motion. Ultrasound
exam reveals a discontinuous tendon. During exploration there is a moderate scar, the oblique pulley is intact, and the end of the FPL are separated by 3 cm.

•What is the next best treatment step?


1. Thumb interphalangeal joint arthrodesis
2. Single-stage flexor pollicis longus reconstruction with palmaris longus graft
3. Silastic tendon implant with staged reconstruction of the flexor pollicus longus repair
4. Tenolysis, partial oblique pulley resection, and primary flexor pollicis longus repair
5. Transfer of the flexor digitorum sublimus from the ring finger to the distal flexor pollicis longus
Question 140
A 40-year-old woman has no active thumb flexion after 2 attempts at repair of her zone II flexor pollicis longus (FPL) laceration during the last 4 months. Extension is also
limited with her wrist and thumb extended, but there is improved interphalangeal extension with the MCP joint flexed. She would like to maintain motion. Ultrasound
exam reveals a discontinuous tendon. During exploration there is a moderate scar, the oblique pulley is intact, and the end of the FPL are separated by 3 cm.

•What is the next best treatment step?


1. Thumb interphalangeal joint arthrodesis
2. Single-stage flexor pollicis longus reconstruction with palmaris longus graft
3. Silastic tendon implant with staged reconstruction of the flexor pollicus longus repair
4. Tenolysis, partial oblique pulley resection, and primary flexor pollicis longus repair
5. Transfer of the flexor digitorum sublimus from the ring finger to the distal flexor pollicis longus
Two-staged reconstruction of FPL tendon

• Precise function of the thumb tip requires IP joint mobility and a functional FPL
musculotendinous unit, therefore arthrodesis would not be warranted.
• After primary repair has failed, staged reconstruction should be attempted.
• First operation- area of the injury is explored and the FPL tendon and scarred
sheath are excised, with preservation of the annular pulleys. Silastic spacer is
implanted from the IP joint to the forearm and fixed to the distal tendon stump
using non-absorbable sutures.
• Second operation- After 8 weeks, the second stage of the tendon graft is
performed typically using a palmaris longus tendon.

Kutsumi K, Amadio PC, Zhao C, Zobitz ME, An KN. Gliding resistance of the flexor pollicis longus
tendon after repair: does partial excision of the oblique pulley affect gliding resistance? Plast Reconstr
Surg. 2006 Nov;118(6):1423-8; discussion 1429-30. PubMed PMID: 17051113.
Unglaub F, Bultmann C, Reiter A, Hahn P. Two-staged reconstruction of the flexor pollicis longus tendon. J
Hand Surg Br. 2006 Aug;31(4):432-5. Epub 2006 Apr 18. PubMed PMID: 16618523.
Question 175
Figure 175 is the lateral radiograph of a 20-year-old man who fell from a roof and has
an isolated injury to his wrist and forearm. His skin is intact. He has moderate pain and
his neurovascular examination reveals a 10-mm 2-point discrimination in median
nerve distribution. Which closed reduction should be
performed in the emergency department?

1. Midcarpal joint dislocation


2. Radioulnar joint dislocation
3. Radiocarpal joint dislocation
4. Radiocarpal and midcarpal joint dislocations
5. Radiocarpal and distal radioulnar joint dislocations
Question 175
Figure 175 is the lateral radiograph of a 20-year-old man who fell from a roof and has
an isolated injury to his wrist and forearm. His skin is intact. He has moderate pain and
his neurovascular examination reveals a 10-mm 2-point discrimination in median
nerve distribution. Which closed reduction should be
performed in the emergency department?

1. Midcarpal joint dislocation


2. Radioulnar joint dislocation
3. Radiocarpal joint dislocation
4. Radiocarpal and midcarpal joint dislocations
5. Radiocarpal and distal radioulnar joint dislocations
Radiocarpal fracture-dislocations

Most common mechanisms are falls from height,


MVA and industrial injuries. Neurologic injury
common with median nerve injury more
common than ulnar. Obtain AP, lateral and
oblique views of the wrist. Moneim classification
(Type 1- no injury to intercarpal articulations; 2-
involve intercarpal injuries). Dumontier
classification (Type 1- radial styloid intact or small
avulsion fx; 2- large styloid fracture fragment).
Treatment includes reduction of the radiocarpal
and DRUJ, stabilization of intercarpal injuries and
repair of osseous avulsions.

Ilyas AM, Mudgal CS. Radiocarpal fracture-dislocations. J Am Acad Orthop Surg. 2008
Nov;16(11):647-55. Review. PubMed PMID: 18978287.
Mudgal CS, Psenica J, Jupiter JB. Radiocarpal fracture-dislocation. J Hand Surg Br. 1999
Feb;24(1):92-8. PubMed PMID: 10190615.
Question 228
Question 228
Explanation
Black et al. Review article, 2011
Dupuytren’s contracture: progressive dz of genetic origin.
Excess myofibroblast proliferation, altered collagen matrix
composition. Thickened/contracted palmar fascia. Pathophys
multifactorial and debated: genetic, trauma, inflam, ischemia,
environ? Tx with open fasciectomy/fasciotomy, perc needle
fasciotomy, injectable collagenase Clostridium histolyticum. Post-op
use static or dynamic extension splinting to minimize cell response
and maximize ROM.

Nydick et al: Retrospective review, 2013.


Patients with Dupuytren’s disease treated by percutaneous
needle fasciotomy and collagenase injection. Clinically successful
treatment in 67% of perc fasciotomy vs 56% of collagenase group.
Satisfaction was similar. Only minor complications (skin tears,
ecchymosis, edema, pruritus, lymphadenopathy). Short term
results.
Question 256
A 65-year-old woman has had right hand numbness and pain in her radial 3 fingers for 12
months. She previously experienced excellent pain relief for 6 weeks after receiving a
carpal tunnel corticosteroid injection. Now the pain wakes her from sleep despite wrist
brace wear. Examination reveals positive Tinel, flexion compression, and Phalen test
findings. What is the most appropriate next treatment step?
1.Splinting
2.Observation
3.Hand therapy
4.Repeat injection
5.Carpal tunnel release
Question 256
A 65-year-old woman has had right hand numbness and pain in her radial 3 fingers for 12
months. She previously experienced excellent pain relief for 6 weeks after receiving a
carpal tunnel corticosteroid injection. Now the pain wakes her from sleep despite wrist
brace wear. Examination reveals positive Tinel, flexion compression, and Phalen test
findings. What is the most appropriate next treatment step?
1.Splinting
2.Observation
3.Hand therapy
4.Repeat injection
5.Carpal tunnel release
• Physical exam for carpal tunnel syndrome
• Phalen’s test: wrist volar flexion for >60 sec produces symptoms
• Tinel’s test: provocative tests performed by tapping the median nerve over
the volar carpal tunne
• Durkan’s test: carpal tunnel compression test; performed by pressing
thumbs over carpal tunnel and holding pressure for 30 seconds; positive if
symptomatic within 30 seconds
• EMG is not needed to establish diagnosis (clinical diagnosis) but
recommended if surgical management is being considered
• Carpal tunnel release indications:
• Failure of nonop tx including steroid injections (if have temporary
improvement with steroid injections is good prognostic indicator for good
result with surgery)
• TAKE HOME: Next step for clinical CTS after failed nonsurgical treatment
is to consider EMG, and perform carpal tunnel release

Graham B. The value added by electrodiagnostic testing in the diagnosis of carpal tunnel syndrome. J Bone Joint Surg Am. 2008 Dec;90(12):2587-93. PubMed

PMID: 19047703.

Rozental TD, LaPorte DM. Hand and wrist reconstruction. In: Flynn JM, ed. Orthopaedic Knowledge Update 10. Rosemont, IL: American Academy of Orthopaedic

Surgeons; 2011:363-376.
Question 263
Question 263
A patient has sustained an extension injury with dorsal proximal
interphalangeal joint dislocation. The joint appears congruent after
reduction, but there is a 15% fracture of the volar tip of the base of the
middle phalanx. The physician should recommend
1. open reduction and internal fixation.
2. no splint and immediate active and passive motion.
3. a dynamic external fixator with early active motion.
4. an extension block splint with active flexion and extension.
5. an intrinsic-plus position splint for 4 weeks (interphalangeal
joints at 0-10 degrees).
Question 263
Question 263
A patient has sustained an extension injury with dorsal proximal
interphalangeal joint dislocation. The joint appears congruent after
reduction, but there is a 15% fracture of the volar tip of the base of the
middle phalanx. The physician should recommend
1. open reduction and internal fixation.
2. no splint and immediate active and passive motion.
3. a dynamic external fixator with early active motion.
4. an extension block splint with active flexion and extension.
5. an intrinsic-plus position splint for 4 weeks (interphalangeal
joints at 0-10 degrees).
Question 263

Volar lip fracture of the middle phalangeal base in dorsal fracture-


dislocation of the PIP joint can be non-operatively managed if there is <
30-35% of articular involvement. The patient may be placed in an
extension block splint to prevent recurrent hyperextension, with early
active mobilization demonstrating good or excellent outcomes in 98% of
cases.

•Gaine WJ, Beardsmore J, Fahmy N. Early active mobilization of volar plate


avulsion fractures. Injury. 1998 Oct;29(8):589-91. PubMed PMID: 10209589.
• McElfresh EC, Dobyns JH, O’Brien ET. Management of fracture-dislocation
of the proximal interphalangeal joints by extension-block splinting. J Bone
Joint Surg Am. 1972 Dec;54(8):1705-11. PubMed PMID: 4653646.
Question 267
•Figure 267 is the clinical photograph of a 36-year-old
woman who has experienced pain and swelling of her left
middle finger; this condition has worsened during the last 3
days. She has received no treatment so far. What is the most
appropriate next step?
–1. Oral antibiotics and close follow up
–2. Aspiration and culture of purulent material
–3. Splint, elevation, and intravenous antibiotics
–4. Urgent surgical drainage through a fish-mouth incision
–5. Urgent surgical drainage through a midaxial incision
Figure 267
Question 267
•Figure 267 is the clinical photograph of a 36-year-old
woman who has experienced pain and swelling of her left
middle finger; this condition has worsened during the last 3
days. She has received no treatment so far. What is the most
appropriate next step?
–1. Oral antibiotics and close follow up
–2. Aspiration and culture of purulent material
–3. Splint, elevation, and intravenous antibiotics
–4. Urgent surgical drainage through a fish-mouth incision
–5. Urgent surgical drainage through a midaxial incision
I&D finger abscess
•Incisions for drainage of felons have been controversial. Fishmouth
incisions have been condemned because of resultant vascular compromise
of the digital pad. Midvolar and high lateral incisions are preferred, with
the point of maximal tenderness guiding placement. Proximal probing
should be avoided because of the risk of inoculation of the flexor sheath.
•Capo JT. Infections. In: Chung KC, Murray PM, eds. Hand Surgery Update V. Rosemont, IL: American Society for Surgery of
the Hand; 2012:391-399.
•Abrams RA, Botte MJ. Hand Infections: Treatment Recommendations for Specific Types. J Am Acad Orthop Surg. 1996
Jul;4(4):219-230. PubMed PMID: 10795057.
Question 271
A 32-year-old mechanic felt a pop in his thumb 4 months after undergoing an
A1 pulley release. He has weakened pinch strength. Figure 271 is a clinical
photograph of his thumb with resisted flexion. The most appropriate
treatment is to reconstruct the

1. Av pulley.
2. A1 pulley.
3. oblique pulley.
4. extensor tendon.
5. abductor pollicis brevis.
Figure 271
Question 271
A 32-year-old mechanic felt a pop in his thumb 4 months after undergoing an
A1 pulley release. He has weakened pinch strength. Figure 271 is a clinical
photograph of his thumb with resisted flexion. The most appropriate
treatment is to reconstruct the

1. Av pulley.
2. A1 pulley.
3. oblique pulley.
4. extensor tendon.
5. abductor pollicis brevis.
Question 271
The oblique pulley is the most important pulley in the thumb. It originates at the proximal half of
the proximal phalanx and fascilitates full excursion of the flexor pollicis longus. It also prevents
bowstringing of the FPL which is why it is important to preserve during trigger thumb release.

•Bayat A, Shaaban H, Giakas G, Lees VC. The pulley system of the thumb: anatomic and biomechanical study. J Hand Surg Am. 2002
Jul;27(4):628-35. PubMed PMID: 12132087.
•Kosiyatrakul A, Jitprapaikulsarn S, Durand S, Oberlin C. Closed flexor pulley rupture of the thumb: case report and review of literature. Hand
Surg. 2009;14(2-3):139-42. PubMed PMID: 20135743.

You might also like